Not certain about Complex Number, Polar Form Question

Click For Summary
The discussion revolves around solving the equation z^3 = 4√2 - 4√2i and expressing the solution in polar form. The initial calculations included finding the modulus and argument, resulting in r = 8 and an angle of -45 degrees. However, the user realizes they need to find the cube roots of the equation, which will yield three distinct solutions. The clarification highlights that the original approach went too far without properly addressing the requirement to solve for z. The conversation emphasizes the importance of understanding the concept of roots in complex numbers.
linuxux
Messages
133
Reaction score
0

Homework Statement


The question:
a)Solve the equation z^{3}=4\sqrt{2}-4\sqrt{2}i..
b)Express the answer in polar form.


The Attempt at a Solution



Here's what i got:

r=\sqrt{\left(4\sqrt{2}\right)^{2}+\left(-4\sqrt{2}\right)^{2}}=8
\tan^{-1}\left(\frac{-4\sqrt{2}}{4\sqrt{2}}\right)=-45^{o}
z^{3}=8^{3}cis\left(-45\cdot3\right)
z^{3}=512\left(cos\frac{5\pi}{4}-i\sin\frac{5\pi}{4}\right)
z^{3}=512\left(\frac{-\sqrt{2}}{2}+\frac{\sqrt{2}}{2}i\right)
=-256\sqrt{2}+256\sqrt{2}i\right)

im not sure if this is right, I am supposed to graph this, and with these kinds of numbers, i figure i went wrong somewhere, i have no idea what they mean by "solve" and how or if this "solves" anything other than the fact that it ends with an equal sign, so if someone can check it for me id appreciate it, thanks."
 
Last edited:
Physics news on Phys.org
i think you are solving for z, so you should have
z^{3}=4\sqrt{2}-4\sqrt{2}i=8 cis\left(-45\right)
and then cube root both sides...
hint: should get 3 roots
 
oh, so i went too far, now i see what I am supposed to be solving, thanks.
 
Question: A clock's minute hand has length 4 and its hour hand has length 3. What is the distance between the tips at the moment when it is increasing most rapidly?(Putnam Exam Question) Answer: Making assumption that both the hands moves at constant angular velocities, the answer is ## \sqrt{7} .## But don't you think this assumption is somewhat doubtful and wrong?

Similar threads

Replies
2
Views
1K
  • · Replies 17 ·
Replies
17
Views
2K
  • · Replies 105 ·
4
Replies
105
Views
6K
Replies
4
Views
2K
  • · Replies 5 ·
Replies
5
Views
2K
  • · Replies 5 ·
Replies
5
Views
1K
Replies
2
Views
2K
  • · Replies 10 ·
Replies
10
Views
2K
  • · Replies 22 ·
Replies
22
Views
910
  • · Replies 4 ·
Replies
4
Views
1K